Line 39: Line 39:
 
a) Why is it necessary that&nbsp;<math>f_k(\lambda) \geq 0</math>&nbsp;for&nbsp;<span class="texhtml">''k'' = 0,1,2</span>?&lt;span style="line-height: 1.5em;" /&gt;  
 
a) Why is it necessary that&nbsp;<math>f_k(\lambda) \geq 0</math>&nbsp;for&nbsp;<span class="texhtml">''k'' = 0,1,2</span>?&lt;span style="line-height: 1.5em;" /&gt;  
  
b) Are the functions, <math> r_0(\lambda) \geq 0<math>, <math>g_0(\lambda) \geq 0<math>, and <math>b_0(\lambda) \geq 0<math>? If so, why? If not, why not?
+
b) Are the functions, <math> r_0(\lambda) \geq 0</math>, <math>g_0(\lambda) \geq 0</math>, and <math>b_0(\lambda) \geq 0</math>? If so, why? If not, why not?
  
c) Derive an formula for the tristimulus vector <math>[r, g, b]^t <math> in terms of the tristimulus vector <math> F=[F_1, F_2, F_3]^t <math>.
+
c) Derive an formula for the tristimulus vector <math>[r, g, b]^t </math> in terms of the tristimulus vector <math> F=[F_1, F_2, F_3]^t </math>.
  
d) Do functions <math> f_k(\lambda) <math> exist, which meet these requirements? If so, give a specific example of such functions.  
+
d) Do functions <math> f_k(\lambda) </math> exist, which meet these requirements? If so, give a specific example of such functions.  
 
[[Category:ECE]] [[Category:QE]] [[Category:CNSIP]] [[Category:Problem_solving]] [[Category:Image_processing]]
 
[[Category:ECE]] [[Category:QE]] [[Category:CNSIP]] [[Category:Problem_solving]] [[Category:Image_processing]]

Revision as of 19:29, 10 November 2014


ECE Ph.D. Qualifying Exam

Communication, Networking, Signal and Image Processing (CS)

Question 5: Image Processing

August 2013



Question

Problem 1. (50 pts)


Problem 2. (50 pts)

Let r0(λ),  g0(λ) , and b0(λ) be the CIE color matching functions for red, green, and blue primaries at 700 nm, 546.1 nm, and 435.8 nm, respectively, and let [r,g,b] be the corresponding CIE tristimulus values. 

Furthermore, let f1(λ)f2(λ), and f3(λ) be the spectral response functions for the three color outputs of a color camera. So for each pixel of the camera sensor, there is a 3-dimensional output vector given by F = [F1,F2,F3]t, where

$ F_1 = \int_{-\infty}^{\infty}f_1(\lambda)I(\lambda)d\lambda $,

$ F_2 = \int_{-\infty}^{\infty}f_2(\lambda)I(\lambda)d\lambda $,

$ F_3 = \int_{-\infty}^{\infty}f_3(\lambda)I(\lambda)d\lambda $

where I(λ) is the energy spectrum of the incoming light and $ f_k(\lambda)\geq 0 $ for k = 0,1,2..

Furthermore, assume there exists a matrix, M, so that


a) Why is it necessary that $ f_k(\lambda) \geq 0 $ for k = 0,1,2?<span style="line-height: 1.5em;" />

b) Are the functions, $ r_0(\lambda) \geq 0 $, $ g_0(\lambda) \geq 0 $, and $ b_0(\lambda) \geq 0 $? If so, why? If not, why not?

c) Derive an formula for the tristimulus vector $ [r, g, b]^t $ in terms of the tristimulus vector $ F=[F_1, F_2, F_3]^t $.

d) Do functions $ f_k(\lambda) $ exist, which meet these requirements? If so, give a specific example of such functions.

Alumni Liaison

BSEE 2004, current Ph.D. student researching signal and image processing.

Landis Huffman